"Lagrangian formulation of electromagetism"의 두 판 사이의 차이

수학노트
둘러보기로 가기 검색하러 가기
imported>Pythagoras0
imported>Pythagoras0
7번째 줄: 7번째 줄:
 
F_{i}=\frac{\partial{L}}{\partial{q^{i}}}=\frac{\partial}{\partial{{q}^{i}}}(eA_{j}\dot{q}^{j})=e\frac{\partial{A_{j}}}{\partial{q}^{i}}\dot{q}^{j}
 
F_{i}=\frac{\partial{L}}{\partial{q^{i}}}=\frac{\partial}{\partial{{q}^{i}}}(eA_{j}\dot{q}^{j})=e\frac{\partial{A_{j}}}{\partial{q}^{i}}\dot{q}^{j}
 
$$
 
$$
*  equation of motion<math>\dot{p}=F</math> Therefore we get
+
*  equation of motion
:<math>m\frac{dv_{i}}{dt}=eF_{ij}\dot{q}^{j}</math>. This is what we call the Lorentz force law.
+
:<math>\dot{p}=F,</math> or
 +
:<math>m\frac{dv_{i}}{dt}=eF_{ij}\dot{q}^{j}.</math>
 +
* This is what we call the Lorentz force law.
 
* force on a particle is same as <math>e\mathbf{E}+e\mathbf{v}\times \mathbf{B}</math>
 
* force on a particle is same as <math>e\mathbf{E}+e\mathbf{v}\times \mathbf{B}</math>
 +
* http://en.wikipedia.org/wiki/Lorentz_force
  
  

2013년 3월 24일 (일) 17:22 판

Lagrangian for a particle

  • Lagrangian for a charged particle in an electromagnetic field \(L=T-V\)

\[L(q,\dot{q})=m||\dot{q}||-e\phi+eA_{i}\dot{q}^{i}\]

  • Euler-Lagrange equations

\[p_{i}=\frac{\partial{L}}{\partial{\dot{q}^{i}}}=m\frac{\dot{q}_{i}}{||\dot{q}_{i}||}+eA_{i}=mv_{i}+eA_{i}\] $$ F_{i}=\frac{\partial{L}}{\partial{q^{i}}}=\frac{\partial}{\partial{{q}^{i}}}(eA_{j}\dot{q}^{j})=e\frac{\partial{A_{j}}}{\partial{q}^{i}}\dot{q}^{j} $$

  • equation of motion

\[\dot{p}=F,\] or \[m\frac{dv_{i}}{dt}=eF_{ij}\dot{q}^{j}.\]


Lagrangian for electromagnetic tensor

free

  • 상호작용이 없는 전자기장의 라그랑지안은 다음과 같다

$$\mathcal{L}_{\text{EM}}= - \frac{1}{4}F_{\mu\nu}F^{\mu\nu}=\frac{1}{2}(\mathbf{E}^2-\mathbf{B}^2)$$ 이 때 \(F_{\mu\nu} = \partial_\mu A_\nu - \partial_\nu A_\mu \,\!\)는 전자기텐서, $A=(A_{\mu})$는 전자기 포텐셜

  • action

\[S=-\frac{1}{4}\int F^{\alpha\beta}F_{\alpha\beta}\,d^{4}x\]

  • 라그랑지안은 전자기 포텐셜의 다음과 같은 변환에 대하여 불변이다

\[A_{\mu}(x) \to A_{\mu}(x)-\partial_{\mu}\Lambda(x)\] 여기서 $\Lambda(x)$는 임의의 스칼라장

  • equation of motion

$$ \partial_\mu F^{\mu\nu}=0 $$

in the presence of $j$ and $\rho$

  • Lagrangian

$$L=-\frac{1}{4}F_{\mu\nu}F^{\mu\nu}-ej_\mu A^\mu$$

  • action

$$S[\phi,A]=\int_{t_1}^{t_2}\int_{\mathbb{R}^3}-\rho\phi+j\cdot A+\frac{\epsilon_0}{2}E^2-\frac{1}{2\mu_0}B^2\,dV\,dt$$

  • w.r.t $\phi$

$$\nabla\cdot E=\frac{\rho}{\epsilon_0}$$

  • w.r.t $A$

$$\nabla\times B=\mu_0j+\epsilon_0\mu_0\frac{\partial E}{\partial t}$$



memo


related items


expositions


blogs


questions